LSAT 90 – Section 4 – Question 15

You need a full course to see this video. Enroll now and get started in less than a minute.

Target time: 1:27

This is question data from the 7Sage LSAT Scorer. You can score your LSATs, track your results, and analyze your performance with pretty charts and vital statistics - all with a Free Account ← sign up in less than 10 seconds

Question
QuickView
Type Tags Answer
Choices
Curve Question
Difficulty
Psg/Game/S
Difficulty
Explanation
PT90 S4 Q15
+LR
Strengthen +Streng
A
49%
162
B
26%
161
C
3%
152
D
7%
154
E
15%
157
139
162
180
+Hardest 148.293 +SubsectionMedium

This is a Strengthening question.

Be careful not to mistake the question stem for a Most Strongly Supported question. In a Strengthening question, you are choosing an answer choice that adds to the support of the argument, while in an MSS question, you are taking the stimulus to be true and finding the answer choice that receives the most support.

Hopefully we got over that hurdle, but there are many more hurdles in the argument itself. You first have to understand how the “even if” is functioning in the stimulus as well as be able to take the contrapositive of the premise. However, this is not just an exercise in formal logic like an SA or a PSA question where you just map out the conditionals. In fact, this is actually more of a phenomenon-hypothesis question where the correct answer choice blocks an alternative hypothesis.

Let’s look at the stimulus. It says you should only buy a frying pan that has a manufacturer's warranty. That sounds like a command. And as if expecting an objection, the author says even if it requires paying a little bit more, I should still go for the warranty. But what if I am not the kind of person that would ever find the warranty form, fill it out, and mail it in? And as if the author had also anticipated this second objection, we are told that even if you would never bother seeking reimbursement should the pan not work, you still need to buy the pan with the warranty.

Do you see what the “even if” is doing? Imagine if the author had said that one should buy a frying pan that has a manufacturer's warranty, full stop. That full stop still completely anticipates any potential objections. So in a sense, the “even if” is not doing much. It is just reinforcing that you do not get out of this rule with these possible objections. If you are buying a frying pan, you'd better buy one with a manufacturer's warranty (buy pan → warranty).

And the premise to this conclusion is that manufacturers will not offer a warranty on a product if doing so means that they will need to reimburse many customers because the product did not work well or last long (reimburse many → no warranty). Let’s take the contrapositive of this. If a manufacturer does offer the warranty, then it is not the case that they need to reimburse many customers. (warranty → not reimburse many). And this is why you should only buy things with manufacturer's warranties. So the unstated assumption in this argument is that a manufacturer's warranty is an indicator of high quality, which means that not having to reimburse many customers is an indicator of high quality.

That is the unstated assumption, but let’s evaluate this assumption. Is it actually the case that not having to reimburse many customers is an indicator of high quality? This is where your phenomenon-hypothesis framework comes in. Let’s say All-Clad, an American manufacturer of frying pans, sells many frying pans but very rarely reimburses customers because of bad products. How do you explain this phenomenon?

One hypothesis might be that they make good products. It works well and lasts long, so not many customers seek reimbursement. And this is what the author presumes. But can you think of an alternative hypothesis? Maybe seeking reimbursement is a difficult process. You have to fill in a thousand different forms and call five different departments. Or maybe the particular market segment for frying pans is just too lazy, like me, to seek reimbursement. Both hypotheses would weaken the argument because now the fact that not very many people call in the warranties is no reflection on product quality.

Correct Answer Choice (A) says that most people who buy a frying pan with a manufacturer's warranty would seek reimbursement should the pan fail to work or last long. This wrecks the hypothesis above. (A) strengthens the argument by knocking out a couple of these competing hypotheses so that the product being high quality is more likely to be the hypothesis that explains the low reimbursements. (A) has identified precisely where the argument is vulnerable and patched it up. It has reaffirmed an assumption the argument already made.

Answer Choice (B) says that all of the frying pans currently on the market covered by warranty work at least as well at the time of purchase as any of the pans not covered by warranty. On first blush, you might think (B) strongly supports the conclusion because it seems to provide another reason to buy the warranty pans. But this is a pretty common trap the LSAT writers use in Strengthening questions, which is to come up with independent reasons to support the conclusion. Unlike (A), (B) does not address any of the premises or unstated assumptions and just independently supports the conclusion.

But if you examine (B) closely, you will see it does not even independently provide you with a reason to buy the pans with warranties. In order for (B) to independently support the conclusion, we need to assume that it is really bad if the non-warranty pans worked better than the warranty pans at the moment of purchase. If we make that assumption, then (B) helps by precluding that possibility. But we don't need to make that assumption. Would it actually be really bad if the non-warranty pans worked better than warranty pans at the moment of purchase? Frying pans are not disposable like plastic forks. Durability matters. So what I care about is whether the non-warranty pan can keep up its quality, because I know if the quality starts degrading on my warranty pan, I can return it. So maybe I care a little bit that non-warranty pans are better than the warranty pans at the time of purchase, but it is not nearly as important as it might initially have seemed.

(B) is analogous to trap answer choices in Weakening questions that look like they are contradicting the conclusion, but upon closer inspection, they do not. This is a Strengthening question so (B)'s trap is to give you something that looks like it is supporting the conclusion, but upon closer inspection, it does not.

Answer Choice (E) is similar to (B). (E) says most frying pans’ manufacturer’s warranties provide for full customer satisfaction. So (E) gives us yet another reason to buy a pan with a warranty. (E) might be a little bit better than (B) because it supports the conclusion more strongly, but (E) still shares the same flaw as (B), which is that it completely ignores the argument. (E) is not entirely irrelevant, just as (B) was not entirely irrelevant in that how good the pan is at the time of purchase surely matters some. But neither (E) nor (B) meaningfully engages with the argument.

Answer Choice (D) says the most expensive frying pans are the ones most likely to work well for many years. But this information is not helpful because we do not know if the most expensive frying pans have a warranty or not. The author only says that one should go for the warranty even if it requires paying more, and never says warranty pans actually do cost more or less.

Answer Choice (C) says the more a frying pan costs, the more likely it is to be covered by a manufacturer's warranty. I suppose (C) is better than (D) because it is at least correlating cost to warranty. But it still does not do much since the author already said to forget the costs. The author thinks a manufacturer's warranty is an indication of the product's quality, so harping on about costs is not going to affect the argument.

Take PrepTest

Review Results

Leave a Reply